$int_{0}^{infty} frac{1}{1 + x^r}:dx = frac{1}{r}Gammaleft( frac{r - 1}{r}right)Gammaleft( frac{1}{r}right)$...












6












$begingroup$



This question already has an answer here:




  • Prove $int_0^{infty}! frac{mathbb{d}x}{1+x^n}=frac{pi}{n sinfrac{pi}{n}}$ using real analysis techniques only

    4 answers




As part of a recent question I posted, I decided to try and generalise for a power of $2$ to any $r in mathbb{R}$. As part of the method I took, I had to solve the following integral:



begin{equation}
I = int_{0}^{infty} frac{1}{1 + x^r}:dx
end{equation}



I believe what I've done is correct, but I'm concerned that I may missed something (in particular whether it holds for all $r neq 0$). So, here I have two questions (1) Is what I've done correct? and (2) What other methods can be employed that doesn't rely on complex analysis?



Here is the method I took:



First make the substitution $u = x^{frac{1}{r}}$ to arrive at



begin{equation}
I = frac{1}{n} int_{0}^{infty} frac{1}{1 + u} cdot u^{1 -frac{1}{r}}:du
end{equation}



We now substitute $t = frac{1}{1 + u}$ to arrive at:



begin{align}
I &= frac{1}{r} int_{1}^{0} t cdot left(frac{1 - t}{t}right)^{frac{1}{r} -1}frac{1}{t^2}:dt = frac{1}{r}int_{0}^{1}t^{-frac{1}{r}}left(1 - tright)^{ frac{1}{r} - 1}:dt \
&= frac{1}{r}Bleft(1 - frac{1}{n}, 1 + frac{1}{r} - 1right) = frac{1}{r} Bleft(frac{r - 1}{r},frac{1}{r}right) \
&= frac{1}{r} Bleft(frac{r - 1}{r},frac{1}{r}right)
end{align}



Wheer $B(a,b)$ is the Beta function. Using the relationship between the Beta and Gamma function we arrive at:



begin{equation}
I = frac{1}{r} frac{Gammaleft( frac{r - 1}{r}right)Gammaleft( frac{1}{r}right)}{Gammaleft(frac{r - 1}{r} + frac{1}{r}right)} = frac{1}{r}Gammaleft( frac{r - 1}{r}right)Gammaleft( frac{1}{r}right)
end{equation}



And so, we arrive at:



begin{equation}
I = int_{0}^{infty} frac{1}{1 + x^r}:dx = frac{1}{r}Gammaleft( frac{r - 1}{r}right)Gammaleft( frac{1}{r}right)
end{equation}



for $r > 1$



As per KemonoChen's comment and others, we can employ Euler's Reflection Formula to position this result for $frac{1}{r} not in mathbb{Z}$ Here, as $r in mathbb{R}, r > 1 rightarrow frac{1}{r} not in mathbb{Z}$ and so our formula holds.



begin{equation}
I = int_{0}^{infty} frac{1}{1 + x^r}:dx = frac{1}{r}Gammaleft( frac{r - 1}{r}right)Gammaleft( frac{1}{r}right) = frac{pi}{rsinleft(frac{pi}{r} right)}
end{equation}



Thank you also to Winther, Jjagmath, and MrTaurho's for their comments and corrections/clarifications.










share|cite|improve this question











$endgroup$



marked as duplicate by Namaste integration
Users with the  integration badge can single-handedly close integration questions as duplicates and reopen them as needed.

StackExchange.ready(function() {
if (StackExchange.options.isMobile) return;

$('.dupe-hammer-message-hover:not(.hover-bound)').each(function() {
var $hover = $(this).addClass('hover-bound'),
$msg = $hover.siblings('.dupe-hammer-message');

$hover.hover(
function() {
$hover.showInfoMessage('', {
messageElement: $msg.clone().show(),
transient: false,
position: { my: 'bottom left', at: 'top center', offsetTop: -7 },
dismissable: false,
relativeToBody: true
});
},
function() {
StackExchange.helpers.removeMessages();
}
);
});
});
Dec 24 '18 at 2:39


This question has been asked before and already has an answer. If those answers do not fully address your question, please ask a new question.














  • 3




    $begingroup$
    You can simplify the $Gamma$ part by using the reflection formula of gamma function.
    $endgroup$
    – Kemono Chen
    Dec 16 '18 at 6:08












  • $begingroup$
    Cheers @KemonoChen!! - Just in the middle of playing the WiiU with my Niece and Nephew - will apply that later and add to the question)
    $endgroup$
    – DavidG
    Dec 16 '18 at 6:16






  • 1




    $begingroup$
    "in particular whether it holds for all $n$": The integral clearly diverges when $n leq 1$
    $endgroup$
    – Winther
    Dec 16 '18 at 10:27








  • 1




    $begingroup$
    For the reflexion formula, in this case you need $1/n notin mathbb{Z}$, which is true for ALL $n>1$ (and, as @Winther observed, this is necessary for the integral to converge)
    $endgroup$
    – jjagmath
    Dec 16 '18 at 11:10






  • 2




    $begingroup$
    @DavidG It's funny that you post a link to the ISO standard. You are right, using $n$ to denote an integer is NOT part of the standard. However, in all the examples of the link, $f$ denotes a function, lowercase letters are used for elements and capital letters for sets, $x$ is used for real numbers, $z$ for complex numbers and, of course, $n$ for integers. ;)
    $endgroup$
    – jjagmath
    Dec 17 '18 at 15:26
















6












$begingroup$



This question already has an answer here:




  • Prove $int_0^{infty}! frac{mathbb{d}x}{1+x^n}=frac{pi}{n sinfrac{pi}{n}}$ using real analysis techniques only

    4 answers




As part of a recent question I posted, I decided to try and generalise for a power of $2$ to any $r in mathbb{R}$. As part of the method I took, I had to solve the following integral:



begin{equation}
I = int_{0}^{infty} frac{1}{1 + x^r}:dx
end{equation}



I believe what I've done is correct, but I'm concerned that I may missed something (in particular whether it holds for all $r neq 0$). So, here I have two questions (1) Is what I've done correct? and (2) What other methods can be employed that doesn't rely on complex analysis?



Here is the method I took:



First make the substitution $u = x^{frac{1}{r}}$ to arrive at



begin{equation}
I = frac{1}{n} int_{0}^{infty} frac{1}{1 + u} cdot u^{1 -frac{1}{r}}:du
end{equation}



We now substitute $t = frac{1}{1 + u}$ to arrive at:



begin{align}
I &= frac{1}{r} int_{1}^{0} t cdot left(frac{1 - t}{t}right)^{frac{1}{r} -1}frac{1}{t^2}:dt = frac{1}{r}int_{0}^{1}t^{-frac{1}{r}}left(1 - tright)^{ frac{1}{r} - 1}:dt \
&= frac{1}{r}Bleft(1 - frac{1}{n}, 1 + frac{1}{r} - 1right) = frac{1}{r} Bleft(frac{r - 1}{r},frac{1}{r}right) \
&= frac{1}{r} Bleft(frac{r - 1}{r},frac{1}{r}right)
end{align}



Wheer $B(a,b)$ is the Beta function. Using the relationship between the Beta and Gamma function we arrive at:



begin{equation}
I = frac{1}{r} frac{Gammaleft( frac{r - 1}{r}right)Gammaleft( frac{1}{r}right)}{Gammaleft(frac{r - 1}{r} + frac{1}{r}right)} = frac{1}{r}Gammaleft( frac{r - 1}{r}right)Gammaleft( frac{1}{r}right)
end{equation}



And so, we arrive at:



begin{equation}
I = int_{0}^{infty} frac{1}{1 + x^r}:dx = frac{1}{r}Gammaleft( frac{r - 1}{r}right)Gammaleft( frac{1}{r}right)
end{equation}



for $r > 1$



As per KemonoChen's comment and others, we can employ Euler's Reflection Formula to position this result for $frac{1}{r} not in mathbb{Z}$ Here, as $r in mathbb{R}, r > 1 rightarrow frac{1}{r} not in mathbb{Z}$ and so our formula holds.



begin{equation}
I = int_{0}^{infty} frac{1}{1 + x^r}:dx = frac{1}{r}Gammaleft( frac{r - 1}{r}right)Gammaleft( frac{1}{r}right) = frac{pi}{rsinleft(frac{pi}{r} right)}
end{equation}



Thank you also to Winther, Jjagmath, and MrTaurho's for their comments and corrections/clarifications.










share|cite|improve this question











$endgroup$



marked as duplicate by Namaste integration
Users with the  integration badge can single-handedly close integration questions as duplicates and reopen them as needed.

StackExchange.ready(function() {
if (StackExchange.options.isMobile) return;

$('.dupe-hammer-message-hover:not(.hover-bound)').each(function() {
var $hover = $(this).addClass('hover-bound'),
$msg = $hover.siblings('.dupe-hammer-message');

$hover.hover(
function() {
$hover.showInfoMessage('', {
messageElement: $msg.clone().show(),
transient: false,
position: { my: 'bottom left', at: 'top center', offsetTop: -7 },
dismissable: false,
relativeToBody: true
});
},
function() {
StackExchange.helpers.removeMessages();
}
);
});
});
Dec 24 '18 at 2:39


This question has been asked before and already has an answer. If those answers do not fully address your question, please ask a new question.














  • 3




    $begingroup$
    You can simplify the $Gamma$ part by using the reflection formula of gamma function.
    $endgroup$
    – Kemono Chen
    Dec 16 '18 at 6:08












  • $begingroup$
    Cheers @KemonoChen!! - Just in the middle of playing the WiiU with my Niece and Nephew - will apply that later and add to the question)
    $endgroup$
    – DavidG
    Dec 16 '18 at 6:16






  • 1




    $begingroup$
    "in particular whether it holds for all $n$": The integral clearly diverges when $n leq 1$
    $endgroup$
    – Winther
    Dec 16 '18 at 10:27








  • 1




    $begingroup$
    For the reflexion formula, in this case you need $1/n notin mathbb{Z}$, which is true for ALL $n>1$ (and, as @Winther observed, this is necessary for the integral to converge)
    $endgroup$
    – jjagmath
    Dec 16 '18 at 11:10






  • 2




    $begingroup$
    @DavidG It's funny that you post a link to the ISO standard. You are right, using $n$ to denote an integer is NOT part of the standard. However, in all the examples of the link, $f$ denotes a function, lowercase letters are used for elements and capital letters for sets, $x$ is used for real numbers, $z$ for complex numbers and, of course, $n$ for integers. ;)
    $endgroup$
    – jjagmath
    Dec 17 '18 at 15:26














6












6








6


4



$begingroup$



This question already has an answer here:




  • Prove $int_0^{infty}! frac{mathbb{d}x}{1+x^n}=frac{pi}{n sinfrac{pi}{n}}$ using real analysis techniques only

    4 answers




As part of a recent question I posted, I decided to try and generalise for a power of $2$ to any $r in mathbb{R}$. As part of the method I took, I had to solve the following integral:



begin{equation}
I = int_{0}^{infty} frac{1}{1 + x^r}:dx
end{equation}



I believe what I've done is correct, but I'm concerned that I may missed something (in particular whether it holds for all $r neq 0$). So, here I have two questions (1) Is what I've done correct? and (2) What other methods can be employed that doesn't rely on complex analysis?



Here is the method I took:



First make the substitution $u = x^{frac{1}{r}}$ to arrive at



begin{equation}
I = frac{1}{n} int_{0}^{infty} frac{1}{1 + u} cdot u^{1 -frac{1}{r}}:du
end{equation}



We now substitute $t = frac{1}{1 + u}$ to arrive at:



begin{align}
I &= frac{1}{r} int_{1}^{0} t cdot left(frac{1 - t}{t}right)^{frac{1}{r} -1}frac{1}{t^2}:dt = frac{1}{r}int_{0}^{1}t^{-frac{1}{r}}left(1 - tright)^{ frac{1}{r} - 1}:dt \
&= frac{1}{r}Bleft(1 - frac{1}{n}, 1 + frac{1}{r} - 1right) = frac{1}{r} Bleft(frac{r - 1}{r},frac{1}{r}right) \
&= frac{1}{r} Bleft(frac{r - 1}{r},frac{1}{r}right)
end{align}



Wheer $B(a,b)$ is the Beta function. Using the relationship between the Beta and Gamma function we arrive at:



begin{equation}
I = frac{1}{r} frac{Gammaleft( frac{r - 1}{r}right)Gammaleft( frac{1}{r}right)}{Gammaleft(frac{r - 1}{r} + frac{1}{r}right)} = frac{1}{r}Gammaleft( frac{r - 1}{r}right)Gammaleft( frac{1}{r}right)
end{equation}



And so, we arrive at:



begin{equation}
I = int_{0}^{infty} frac{1}{1 + x^r}:dx = frac{1}{r}Gammaleft( frac{r - 1}{r}right)Gammaleft( frac{1}{r}right)
end{equation}



for $r > 1$



As per KemonoChen's comment and others, we can employ Euler's Reflection Formula to position this result for $frac{1}{r} not in mathbb{Z}$ Here, as $r in mathbb{R}, r > 1 rightarrow frac{1}{r} not in mathbb{Z}$ and so our formula holds.



begin{equation}
I = int_{0}^{infty} frac{1}{1 + x^r}:dx = frac{1}{r}Gammaleft( frac{r - 1}{r}right)Gammaleft( frac{1}{r}right) = frac{pi}{rsinleft(frac{pi}{r} right)}
end{equation}



Thank you also to Winther, Jjagmath, and MrTaurho's for their comments and corrections/clarifications.










share|cite|improve this question











$endgroup$





This question already has an answer here:




  • Prove $int_0^{infty}! frac{mathbb{d}x}{1+x^n}=frac{pi}{n sinfrac{pi}{n}}$ using real analysis techniques only

    4 answers




As part of a recent question I posted, I decided to try and generalise for a power of $2$ to any $r in mathbb{R}$. As part of the method I took, I had to solve the following integral:



begin{equation}
I = int_{0}^{infty} frac{1}{1 + x^r}:dx
end{equation}



I believe what I've done is correct, but I'm concerned that I may missed something (in particular whether it holds for all $r neq 0$). So, here I have two questions (1) Is what I've done correct? and (2) What other methods can be employed that doesn't rely on complex analysis?



Here is the method I took:



First make the substitution $u = x^{frac{1}{r}}$ to arrive at



begin{equation}
I = frac{1}{n} int_{0}^{infty} frac{1}{1 + u} cdot u^{1 -frac{1}{r}}:du
end{equation}



We now substitute $t = frac{1}{1 + u}$ to arrive at:



begin{align}
I &= frac{1}{r} int_{1}^{0} t cdot left(frac{1 - t}{t}right)^{frac{1}{r} -1}frac{1}{t^2}:dt = frac{1}{r}int_{0}^{1}t^{-frac{1}{r}}left(1 - tright)^{ frac{1}{r} - 1}:dt \
&= frac{1}{r}Bleft(1 - frac{1}{n}, 1 + frac{1}{r} - 1right) = frac{1}{r} Bleft(frac{r - 1}{r},frac{1}{r}right) \
&= frac{1}{r} Bleft(frac{r - 1}{r},frac{1}{r}right)
end{align}



Wheer $B(a,b)$ is the Beta function. Using the relationship between the Beta and Gamma function we arrive at:



begin{equation}
I = frac{1}{r} frac{Gammaleft( frac{r - 1}{r}right)Gammaleft( frac{1}{r}right)}{Gammaleft(frac{r - 1}{r} + frac{1}{r}right)} = frac{1}{r}Gammaleft( frac{r - 1}{r}right)Gammaleft( frac{1}{r}right)
end{equation}



And so, we arrive at:



begin{equation}
I = int_{0}^{infty} frac{1}{1 + x^r}:dx = frac{1}{r}Gammaleft( frac{r - 1}{r}right)Gammaleft( frac{1}{r}right)
end{equation}



for $r > 1$



As per KemonoChen's comment and others, we can employ Euler's Reflection Formula to position this result for $frac{1}{r} not in mathbb{Z}$ Here, as $r in mathbb{R}, r > 1 rightarrow frac{1}{r} not in mathbb{Z}$ and so our formula holds.



begin{equation}
I = int_{0}^{infty} frac{1}{1 + x^r}:dx = frac{1}{r}Gammaleft( frac{r - 1}{r}right)Gammaleft( frac{1}{r}right) = frac{pi}{rsinleft(frac{pi}{r} right)}
end{equation}



Thank you also to Winther, Jjagmath, and MrTaurho's for their comments and corrections/clarifications.





This question already has an answer here:




  • Prove $int_0^{infty}! frac{mathbb{d}x}{1+x^n}=frac{pi}{n sinfrac{pi}{n}}$ using real analysis techniques only

    4 answers








real-analysis integration definite-integrals big-list






share|cite|improve this question















share|cite|improve this question













share|cite|improve this question




share|cite|improve this question








edited Dec 16 '18 at 23:27







DavidG

















asked Dec 16 '18 at 6:07









DavidGDavidG

2,5461726




2,5461726




marked as duplicate by Namaste integration
Users with the  integration badge can single-handedly close integration questions as duplicates and reopen them as needed.

StackExchange.ready(function() {
if (StackExchange.options.isMobile) return;

$('.dupe-hammer-message-hover:not(.hover-bound)').each(function() {
var $hover = $(this).addClass('hover-bound'),
$msg = $hover.siblings('.dupe-hammer-message');

$hover.hover(
function() {
$hover.showInfoMessage('', {
messageElement: $msg.clone().show(),
transient: false,
position: { my: 'bottom left', at: 'top center', offsetTop: -7 },
dismissable: false,
relativeToBody: true
});
},
function() {
StackExchange.helpers.removeMessages();
}
);
});
});
Dec 24 '18 at 2:39


This question has been asked before and already has an answer. If those answers do not fully address your question, please ask a new question.









marked as duplicate by Namaste integration
Users with the  integration badge can single-handedly close integration questions as duplicates and reopen them as needed.

StackExchange.ready(function() {
if (StackExchange.options.isMobile) return;

$('.dupe-hammer-message-hover:not(.hover-bound)').each(function() {
var $hover = $(this).addClass('hover-bound'),
$msg = $hover.siblings('.dupe-hammer-message');

$hover.hover(
function() {
$hover.showInfoMessage('', {
messageElement: $msg.clone().show(),
transient: false,
position: { my: 'bottom left', at: 'top center', offsetTop: -7 },
dismissable: false,
relativeToBody: true
});
},
function() {
StackExchange.helpers.removeMessages();
}
);
});
});
Dec 24 '18 at 2:39


This question has been asked before and already has an answer. If those answers do not fully address your question, please ask a new question.










  • 3




    $begingroup$
    You can simplify the $Gamma$ part by using the reflection formula of gamma function.
    $endgroup$
    – Kemono Chen
    Dec 16 '18 at 6:08












  • $begingroup$
    Cheers @KemonoChen!! - Just in the middle of playing the WiiU with my Niece and Nephew - will apply that later and add to the question)
    $endgroup$
    – DavidG
    Dec 16 '18 at 6:16






  • 1




    $begingroup$
    "in particular whether it holds for all $n$": The integral clearly diverges when $n leq 1$
    $endgroup$
    – Winther
    Dec 16 '18 at 10:27








  • 1




    $begingroup$
    For the reflexion formula, in this case you need $1/n notin mathbb{Z}$, which is true for ALL $n>1$ (and, as @Winther observed, this is necessary for the integral to converge)
    $endgroup$
    – jjagmath
    Dec 16 '18 at 11:10






  • 2




    $begingroup$
    @DavidG It's funny that you post a link to the ISO standard. You are right, using $n$ to denote an integer is NOT part of the standard. However, in all the examples of the link, $f$ denotes a function, lowercase letters are used for elements and capital letters for sets, $x$ is used for real numbers, $z$ for complex numbers and, of course, $n$ for integers. ;)
    $endgroup$
    – jjagmath
    Dec 17 '18 at 15:26














  • 3




    $begingroup$
    You can simplify the $Gamma$ part by using the reflection formula of gamma function.
    $endgroup$
    – Kemono Chen
    Dec 16 '18 at 6:08












  • $begingroup$
    Cheers @KemonoChen!! - Just in the middle of playing the WiiU with my Niece and Nephew - will apply that later and add to the question)
    $endgroup$
    – DavidG
    Dec 16 '18 at 6:16






  • 1




    $begingroup$
    "in particular whether it holds for all $n$": The integral clearly diverges when $n leq 1$
    $endgroup$
    – Winther
    Dec 16 '18 at 10:27








  • 1




    $begingroup$
    For the reflexion formula, in this case you need $1/n notin mathbb{Z}$, which is true for ALL $n>1$ (and, as @Winther observed, this is necessary for the integral to converge)
    $endgroup$
    – jjagmath
    Dec 16 '18 at 11:10






  • 2




    $begingroup$
    @DavidG It's funny that you post a link to the ISO standard. You are right, using $n$ to denote an integer is NOT part of the standard. However, in all the examples of the link, $f$ denotes a function, lowercase letters are used for elements and capital letters for sets, $x$ is used for real numbers, $z$ for complex numbers and, of course, $n$ for integers. ;)
    $endgroup$
    – jjagmath
    Dec 17 '18 at 15:26








3




3




$begingroup$
You can simplify the $Gamma$ part by using the reflection formula of gamma function.
$endgroup$
– Kemono Chen
Dec 16 '18 at 6:08






$begingroup$
You can simplify the $Gamma$ part by using the reflection formula of gamma function.
$endgroup$
– Kemono Chen
Dec 16 '18 at 6:08














$begingroup$
Cheers @KemonoChen!! - Just in the middle of playing the WiiU with my Niece and Nephew - will apply that later and add to the question)
$endgroup$
– DavidG
Dec 16 '18 at 6:16




$begingroup$
Cheers @KemonoChen!! - Just in the middle of playing the WiiU with my Niece and Nephew - will apply that later and add to the question)
$endgroup$
– DavidG
Dec 16 '18 at 6:16




1




1




$begingroup$
"in particular whether it holds for all $n$": The integral clearly diverges when $n leq 1$
$endgroup$
– Winther
Dec 16 '18 at 10:27






$begingroup$
"in particular whether it holds for all $n$": The integral clearly diverges when $n leq 1$
$endgroup$
– Winther
Dec 16 '18 at 10:27






1




1




$begingroup$
For the reflexion formula, in this case you need $1/n notin mathbb{Z}$, which is true for ALL $n>1$ (and, as @Winther observed, this is necessary for the integral to converge)
$endgroup$
– jjagmath
Dec 16 '18 at 11:10




$begingroup$
For the reflexion formula, in this case you need $1/n notin mathbb{Z}$, which is true for ALL $n>1$ (and, as @Winther observed, this is necessary for the integral to converge)
$endgroup$
– jjagmath
Dec 16 '18 at 11:10




2




2




$begingroup$
@DavidG It's funny that you post a link to the ISO standard. You are right, using $n$ to denote an integer is NOT part of the standard. However, in all the examples of the link, $f$ denotes a function, lowercase letters are used for elements and capital letters for sets, $x$ is used for real numbers, $z$ for complex numbers and, of course, $n$ for integers. ;)
$endgroup$
– jjagmath
Dec 17 '18 at 15:26




$begingroup$
@DavidG It's funny that you post a link to the ISO standard. You are right, using $n$ to denote an integer is NOT part of the standard. However, in all the examples of the link, $f$ denotes a function, lowercase letters are used for elements and capital letters for sets, $x$ is used for real numbers, $z$ for complex numbers and, of course, $n$ for integers. ;)
$endgroup$
– jjagmath
Dec 17 '18 at 15:26










3 Answers
3






active

oldest

votes


















9












$begingroup$

Your given integral is closely related to the Mellin transform and can be evaluated by using Ramanujan's Master Theorem.




Ramanujan's Master Theorem



Let $f(x)$ be an analytic function with a MacLaurin Expansion of the form
$$f(x)=sum_{k=0}^{infty}frac{phi(k)}{k!}(-x)^k$$then the Mellin Transform of this function is given by
$$int_0^{infty}x^{s-1}f(x)dx=Gamma(s)phi(-s)$$




In order to get there we can expand the fraction as a geometric series



$$begin{align}
I=int_0^{infty}frac1{1+x^n}dx&=int_0^{infty}sum_{k=0}^{infty}(-1)^k x^{kn}dx
end{align}$$



Now by applying the substitution $t=x^n$ followed by a little bit of reshaping yields to



$$begin{align}
I&=int_0^{infty}sum_{k=0}^{infty}(-1)^k x^{kn}dx\
&=frac1nint_0^{infty}t^{1/n-1}sum_{k=0}^{infty}(-1)^kfrac{k!}{k!}t^{k}dt\
&=frac1nint_0^{infty}t^{1/n-1}sum_{k=0}^{infty}(-1)^kfrac{phi(k)}{k!}t^{k}dt
end{align}$$



Now we can apply Ramanujan's Master Theorem with $s=1/n$ and $phi(k)=Gamma(k+1)$ to get



$$begin{align}
I=frac1nint_0^{infty}t^{1/n-1}sum_{k=0}^{infty}(-1)^kfrac{phi(k)}{k!}t^{k}dt&=frac1nGammaleft(frac1nright)Gammaleft(1-frac1nright)
end{align}$$



And finally this can be simplified, as already pointed out, by using Euler's Reflection Formula $($as long as it holds i.e. for all $1/nnotinmathbb Z$$)$ to get




$$I=int_0^{infty}frac1{1+x^n}dx=frac1nfrac{pi}{sinleft(frac{pi}{n}right)}$$







share|cite|improve this answer











$endgroup$









  • 4




    $begingroup$
    This is my first dance with Ramanujan's Master Theorem. Only last week I was blown away learning 'Glassers Master Theorem'! now this! two mathgasms in two weeks! ..... Thanks for the post!
    $endgroup$
    – DavidG
    Dec 17 '18 at 8:42






  • 2




    $begingroup$
    @DavidG I appreciate that I have sparked you in favor of Ramanujan's Master Theorem. To be honest it is my favourite Theorem right now due the capability of it $($seefor example [here](math.stackexchange.com/q/3012122), [here](math.stackexchange.com/q/2937122) and [here](math.stackexchange.com/questions/2945796/….
    $endgroup$
    – mrtaurho
    Dec 17 '18 at 20:05












  • $begingroup$
    In light of spending the last week absorbed with tricky series and integrals, Ramanujan's Master Theorem seems almost too good to be true. We are certainly very lucky to have his contributions.
    $endgroup$
    – DavidG
    Dec 19 '18 at 4:22










  • $begingroup$
    @DavidG But it is true! ^^ One of the most asthonishing proofs of Ramanujan's Master Theorem for me was the one proposed by the user Count Iblis here using a technique known as "Umbral calculus". Moreover he also gave another possible proof which does not invoke the Mellin Inversion Theorem aswell.
    $endgroup$
    – mrtaurho
    Dec 19 '18 at 12:15



















3












$begingroup$

Once again I will offer up a method that first converts the integral to a double integral.



For $r > 0$, we begin by enforcing a substitution of $x mapsto x^{1/r}$. Doing so yields
$$I = frac{1}{r} int_0^infty frac{x^{1/r - 1}}{1 + x} , dx.$$



Now noting that
$$frac{1}{1 + x} = int_0^infty e^{-u(1 + x)} , du,$$
our integral can be rewritten as
$$I = frac{1}{r} int_0^infty x^{1/r - 1} int_0^infty e^{-u (1 + x)} , du , dx,$$
or
$$I = frac{1}{r} int_0^infty e^{-u} int_0^infty x^{1/r - 1} e^{-ux} , dx , du,$$
after changing the order of integration.



Next we enforce a substitution of $x mapsto x/u$. This gives
begin{align}
I &= frac{1}{r} int_0^infty u^{- 1/r} e^{-u} , du int_0^infty x^{1/r - 1} e^{-x} , dx\
&= frac{1}{r} Gamma left (1 - frac{1}{r} right ) Gamma left (frac{1}{r} right )\
&= frac{pi}{r sin left (frac{pi}{r} right )},
end{align}

where in the last line we have made use of Euler's reflexion formula for the gamma function.






share|cite|improve this answer









$endgroup$









  • 2




    $begingroup$
    You and your double integrals!!!!!!!! no, love it mate, thanks heaps. Great to see it used in many ways.
    $endgroup$
    – DavidG
    Dec 17 '18 at 9:26



















3












$begingroup$

NOT A FULL SOLUTION:



I've been working with special cases of the integral.



Here we will consider $r = 2m$ where $m in mathbb{N}$. In doing so, we observe that the roots of the denominator are $m$ pairs of complex roots $(z, c(z))$ where $c(z)$ is the conjugate of $z$. To verify this:



begin{align}
x^{2m} + 1 = 0 rightarrow x^{2m} = e^{pi i}
end{align}



By De Moivre's formula, we observe that:



begin{align}
x = expleft({frac{pi + 2pi j}{2m} i} right) mbox{ for } j = 0dots 2m - 1
end{align}



Which we can express as the set



begin{align}
S &= Bigg{ expleft({frac{pi + 2pi cdot 0}{2m} i} right) , :expleft({frac{pi + 2pi cdot 1}{2m} i} right),dots,:expleft({frac{pi + 2pi cdot (2m - 2)}{2m} i} right)\
&qquad:expleft({frac{pi + 2pi cdot (2m - 1)}{2m} i} right)Bigg} \
end{align}



Which can be expressed as the set of 2-tuples



begin{align}
S &= left{ left( expleft({frac{pi + 2pi j}{2m} i} right) , :expleft({frac{pi + 2pi(2m - 1 - j )}{2m} i} right)right): bigg|: j = 0 dots m - 1right}\
& = left{ (z_j, cleft(z_jright):|: j = 0 dots m - 1 right}
end{align}



From here, we can factor $x^{2m} + 1$ into the form



begin{align}
x^{2m} + 1 &= prod_{r in S} left(x + r_jright)left(x + c(r_j)right) \
&= prod_{r in S} left(x^2 + left(r_j + c(r_j)right)x + r_j c(r_j)right) \
&= prod_{r in S} left(x^2 + 2Releft(r_jright)x + left|r_j right|^2right)
end{align}



For our case here $left|r_j right|^2 = 1$ and $Releft(r_jright) = cosleft({frac{pi + 2pi j}{2m} } right)$. Hence,



begin{align}
frac{1}{x^{2m} + 1} = prod_{j = 0}^{m - 1}frac{1}{ x^2 + 2cosleft({frac{pi + 2pi j}{2m} } right)x + 1}
end{align}



From here, to evaluate the integral we must employ Partial Fraction Decomposition:



begin{align}
frac{1}{x^{2m} + 1} = prod_{j = 0}^{m - 1}frac{1}{ x^2 + 2cosleft({frac{pi + 2pi j}{2m} } right)x + 1} = sum_{j = 0}^{m - 1}frac{alpha_j + beta_jx}{ x^2 + 2cosleft({frac{pi + 2pi j}{2m} } right)x + 1}
end{align}



And solve for $alpha_j$ and $beta_j$. Putting the coefficents to the side we can find general expressions for the integral:



begin{align}
frac{1}{x^{2m} + 1} = prod_{j = 0}^{m - 1}frac{1}{ x^2 + 2cosleft({frac{pi + 2pi j}{2m} } right)x + 1}
end{align}



From here, to evaluate the integral we must employ Partial Fraction Decomposition:



begin{align}
int_{0}^{infty}frac{1}{x^{2m} + 1}:dx &= int_{0}^{infty}sum_{j = 0}^{m - 1}frac{alpha_j + beta_jx}{ x^2 + 2cosleft({frac{pi + 2pi j}{2m} } right)x + 1}:dx \
&= sum_{j = 0}^{m - 1}left[ int_{0}^{infty}frac{alpha_j}{ x^2 + 2cosleft({frac{pi + 2pi j}{2m} } right)x + 1}:dx + int_{0}^{infty}frac{beta_jx}{ x^2 + 2cosleft({frac{pi + 2pi j}{2m} } right)x + 1}:dxright] \
&=sum_{j = 0}^{m - 1}left[ I_1 + I_2right]
end{align}



Evaluating each individually:
begin{align}
int_{0}^{infty} frac{alpha_j}{ x^2 + 2cosleft(frac{pi + 2pi j}{2m} right)x + 1}:dx &= left[ cscleft(frac{pi + 2pi j}{2m} right)arctanleft(frac{(x - 1)tanleft(frac{pi + 2pi j}{4m} right)}{x + 1} right)right]_{0}^{infty} \
&= cscleft(frac{pi + 2pi j}{2m} right)left( frac{pi + 2pi j}{2m}right)
end{align}



Now if we consider the second integral on it's own we find that as a general expression the integral doesn't converge. This doesn't take away from this method, but to proceed we must solve for the unknown coefficients to proceed.






share|cite|improve this answer











$endgroup$




















    3 Answers
    3






    active

    oldest

    votes








    3 Answers
    3






    active

    oldest

    votes









    active

    oldest

    votes






    active

    oldest

    votes









    9












    $begingroup$

    Your given integral is closely related to the Mellin transform and can be evaluated by using Ramanujan's Master Theorem.




    Ramanujan's Master Theorem



    Let $f(x)$ be an analytic function with a MacLaurin Expansion of the form
    $$f(x)=sum_{k=0}^{infty}frac{phi(k)}{k!}(-x)^k$$then the Mellin Transform of this function is given by
    $$int_0^{infty}x^{s-1}f(x)dx=Gamma(s)phi(-s)$$




    In order to get there we can expand the fraction as a geometric series



    $$begin{align}
    I=int_0^{infty}frac1{1+x^n}dx&=int_0^{infty}sum_{k=0}^{infty}(-1)^k x^{kn}dx
    end{align}$$



    Now by applying the substitution $t=x^n$ followed by a little bit of reshaping yields to



    $$begin{align}
    I&=int_0^{infty}sum_{k=0}^{infty}(-1)^k x^{kn}dx\
    &=frac1nint_0^{infty}t^{1/n-1}sum_{k=0}^{infty}(-1)^kfrac{k!}{k!}t^{k}dt\
    &=frac1nint_0^{infty}t^{1/n-1}sum_{k=0}^{infty}(-1)^kfrac{phi(k)}{k!}t^{k}dt
    end{align}$$



    Now we can apply Ramanujan's Master Theorem with $s=1/n$ and $phi(k)=Gamma(k+1)$ to get



    $$begin{align}
    I=frac1nint_0^{infty}t^{1/n-1}sum_{k=0}^{infty}(-1)^kfrac{phi(k)}{k!}t^{k}dt&=frac1nGammaleft(frac1nright)Gammaleft(1-frac1nright)
    end{align}$$



    And finally this can be simplified, as already pointed out, by using Euler's Reflection Formula $($as long as it holds i.e. for all $1/nnotinmathbb Z$$)$ to get




    $$I=int_0^{infty}frac1{1+x^n}dx=frac1nfrac{pi}{sinleft(frac{pi}{n}right)}$$







    share|cite|improve this answer











    $endgroup$









    • 4




      $begingroup$
      This is my first dance with Ramanujan's Master Theorem. Only last week I was blown away learning 'Glassers Master Theorem'! now this! two mathgasms in two weeks! ..... Thanks for the post!
      $endgroup$
      – DavidG
      Dec 17 '18 at 8:42






    • 2




      $begingroup$
      @DavidG I appreciate that I have sparked you in favor of Ramanujan's Master Theorem. To be honest it is my favourite Theorem right now due the capability of it $($seefor example [here](math.stackexchange.com/q/3012122), [here](math.stackexchange.com/q/2937122) and [here](math.stackexchange.com/questions/2945796/….
      $endgroup$
      – mrtaurho
      Dec 17 '18 at 20:05












    • $begingroup$
      In light of spending the last week absorbed with tricky series and integrals, Ramanujan's Master Theorem seems almost too good to be true. We are certainly very lucky to have his contributions.
      $endgroup$
      – DavidG
      Dec 19 '18 at 4:22










    • $begingroup$
      @DavidG But it is true! ^^ One of the most asthonishing proofs of Ramanujan's Master Theorem for me was the one proposed by the user Count Iblis here using a technique known as "Umbral calculus". Moreover he also gave another possible proof which does not invoke the Mellin Inversion Theorem aswell.
      $endgroup$
      – mrtaurho
      Dec 19 '18 at 12:15
















    9












    $begingroup$

    Your given integral is closely related to the Mellin transform and can be evaluated by using Ramanujan's Master Theorem.




    Ramanujan's Master Theorem



    Let $f(x)$ be an analytic function with a MacLaurin Expansion of the form
    $$f(x)=sum_{k=0}^{infty}frac{phi(k)}{k!}(-x)^k$$then the Mellin Transform of this function is given by
    $$int_0^{infty}x^{s-1}f(x)dx=Gamma(s)phi(-s)$$




    In order to get there we can expand the fraction as a geometric series



    $$begin{align}
    I=int_0^{infty}frac1{1+x^n}dx&=int_0^{infty}sum_{k=0}^{infty}(-1)^k x^{kn}dx
    end{align}$$



    Now by applying the substitution $t=x^n$ followed by a little bit of reshaping yields to



    $$begin{align}
    I&=int_0^{infty}sum_{k=0}^{infty}(-1)^k x^{kn}dx\
    &=frac1nint_0^{infty}t^{1/n-1}sum_{k=0}^{infty}(-1)^kfrac{k!}{k!}t^{k}dt\
    &=frac1nint_0^{infty}t^{1/n-1}sum_{k=0}^{infty}(-1)^kfrac{phi(k)}{k!}t^{k}dt
    end{align}$$



    Now we can apply Ramanujan's Master Theorem with $s=1/n$ and $phi(k)=Gamma(k+1)$ to get



    $$begin{align}
    I=frac1nint_0^{infty}t^{1/n-1}sum_{k=0}^{infty}(-1)^kfrac{phi(k)}{k!}t^{k}dt&=frac1nGammaleft(frac1nright)Gammaleft(1-frac1nright)
    end{align}$$



    And finally this can be simplified, as already pointed out, by using Euler's Reflection Formula $($as long as it holds i.e. for all $1/nnotinmathbb Z$$)$ to get




    $$I=int_0^{infty}frac1{1+x^n}dx=frac1nfrac{pi}{sinleft(frac{pi}{n}right)}$$







    share|cite|improve this answer











    $endgroup$









    • 4




      $begingroup$
      This is my first dance with Ramanujan's Master Theorem. Only last week I was blown away learning 'Glassers Master Theorem'! now this! two mathgasms in two weeks! ..... Thanks for the post!
      $endgroup$
      – DavidG
      Dec 17 '18 at 8:42






    • 2




      $begingroup$
      @DavidG I appreciate that I have sparked you in favor of Ramanujan's Master Theorem. To be honest it is my favourite Theorem right now due the capability of it $($seefor example [here](math.stackexchange.com/q/3012122), [here](math.stackexchange.com/q/2937122) and [here](math.stackexchange.com/questions/2945796/….
      $endgroup$
      – mrtaurho
      Dec 17 '18 at 20:05












    • $begingroup$
      In light of spending the last week absorbed with tricky series and integrals, Ramanujan's Master Theorem seems almost too good to be true. We are certainly very lucky to have his contributions.
      $endgroup$
      – DavidG
      Dec 19 '18 at 4:22










    • $begingroup$
      @DavidG But it is true! ^^ One of the most asthonishing proofs of Ramanujan's Master Theorem for me was the one proposed by the user Count Iblis here using a technique known as "Umbral calculus". Moreover he also gave another possible proof which does not invoke the Mellin Inversion Theorem aswell.
      $endgroup$
      – mrtaurho
      Dec 19 '18 at 12:15














    9












    9








    9





    $begingroup$

    Your given integral is closely related to the Mellin transform and can be evaluated by using Ramanujan's Master Theorem.




    Ramanujan's Master Theorem



    Let $f(x)$ be an analytic function with a MacLaurin Expansion of the form
    $$f(x)=sum_{k=0}^{infty}frac{phi(k)}{k!}(-x)^k$$then the Mellin Transform of this function is given by
    $$int_0^{infty}x^{s-1}f(x)dx=Gamma(s)phi(-s)$$




    In order to get there we can expand the fraction as a geometric series



    $$begin{align}
    I=int_0^{infty}frac1{1+x^n}dx&=int_0^{infty}sum_{k=0}^{infty}(-1)^k x^{kn}dx
    end{align}$$



    Now by applying the substitution $t=x^n$ followed by a little bit of reshaping yields to



    $$begin{align}
    I&=int_0^{infty}sum_{k=0}^{infty}(-1)^k x^{kn}dx\
    &=frac1nint_0^{infty}t^{1/n-1}sum_{k=0}^{infty}(-1)^kfrac{k!}{k!}t^{k}dt\
    &=frac1nint_0^{infty}t^{1/n-1}sum_{k=0}^{infty}(-1)^kfrac{phi(k)}{k!}t^{k}dt
    end{align}$$



    Now we can apply Ramanujan's Master Theorem with $s=1/n$ and $phi(k)=Gamma(k+1)$ to get



    $$begin{align}
    I=frac1nint_0^{infty}t^{1/n-1}sum_{k=0}^{infty}(-1)^kfrac{phi(k)}{k!}t^{k}dt&=frac1nGammaleft(frac1nright)Gammaleft(1-frac1nright)
    end{align}$$



    And finally this can be simplified, as already pointed out, by using Euler's Reflection Formula $($as long as it holds i.e. for all $1/nnotinmathbb Z$$)$ to get




    $$I=int_0^{infty}frac1{1+x^n}dx=frac1nfrac{pi}{sinleft(frac{pi}{n}right)}$$







    share|cite|improve this answer











    $endgroup$



    Your given integral is closely related to the Mellin transform and can be evaluated by using Ramanujan's Master Theorem.




    Ramanujan's Master Theorem



    Let $f(x)$ be an analytic function with a MacLaurin Expansion of the form
    $$f(x)=sum_{k=0}^{infty}frac{phi(k)}{k!}(-x)^k$$then the Mellin Transform of this function is given by
    $$int_0^{infty}x^{s-1}f(x)dx=Gamma(s)phi(-s)$$




    In order to get there we can expand the fraction as a geometric series



    $$begin{align}
    I=int_0^{infty}frac1{1+x^n}dx&=int_0^{infty}sum_{k=0}^{infty}(-1)^k x^{kn}dx
    end{align}$$



    Now by applying the substitution $t=x^n$ followed by a little bit of reshaping yields to



    $$begin{align}
    I&=int_0^{infty}sum_{k=0}^{infty}(-1)^k x^{kn}dx\
    &=frac1nint_0^{infty}t^{1/n-1}sum_{k=0}^{infty}(-1)^kfrac{k!}{k!}t^{k}dt\
    &=frac1nint_0^{infty}t^{1/n-1}sum_{k=0}^{infty}(-1)^kfrac{phi(k)}{k!}t^{k}dt
    end{align}$$



    Now we can apply Ramanujan's Master Theorem with $s=1/n$ and $phi(k)=Gamma(k+1)$ to get



    $$begin{align}
    I=frac1nint_0^{infty}t^{1/n-1}sum_{k=0}^{infty}(-1)^kfrac{phi(k)}{k!}t^{k}dt&=frac1nGammaleft(frac1nright)Gammaleft(1-frac1nright)
    end{align}$$



    And finally this can be simplified, as already pointed out, by using Euler's Reflection Formula $($as long as it holds i.e. for all $1/nnotinmathbb Z$$)$ to get




    $$I=int_0^{infty}frac1{1+x^n}dx=frac1nfrac{pi}{sinleft(frac{pi}{n}right)}$$








    share|cite|improve this answer














    share|cite|improve this answer



    share|cite|improve this answer








    edited Jan 8 at 10:20

























    answered Dec 16 '18 at 11:10









    mrtaurhomrtaurho

    5,74551540




    5,74551540








    • 4




      $begingroup$
      This is my first dance with Ramanujan's Master Theorem. Only last week I was blown away learning 'Glassers Master Theorem'! now this! two mathgasms in two weeks! ..... Thanks for the post!
      $endgroup$
      – DavidG
      Dec 17 '18 at 8:42






    • 2




      $begingroup$
      @DavidG I appreciate that I have sparked you in favor of Ramanujan's Master Theorem. To be honest it is my favourite Theorem right now due the capability of it $($seefor example [here](math.stackexchange.com/q/3012122), [here](math.stackexchange.com/q/2937122) and [here](math.stackexchange.com/questions/2945796/….
      $endgroup$
      – mrtaurho
      Dec 17 '18 at 20:05












    • $begingroup$
      In light of spending the last week absorbed with tricky series and integrals, Ramanujan's Master Theorem seems almost too good to be true. We are certainly very lucky to have his contributions.
      $endgroup$
      – DavidG
      Dec 19 '18 at 4:22










    • $begingroup$
      @DavidG But it is true! ^^ One of the most asthonishing proofs of Ramanujan's Master Theorem for me was the one proposed by the user Count Iblis here using a technique known as "Umbral calculus". Moreover he also gave another possible proof which does not invoke the Mellin Inversion Theorem aswell.
      $endgroup$
      – mrtaurho
      Dec 19 '18 at 12:15














    • 4




      $begingroup$
      This is my first dance with Ramanujan's Master Theorem. Only last week I was blown away learning 'Glassers Master Theorem'! now this! two mathgasms in two weeks! ..... Thanks for the post!
      $endgroup$
      – DavidG
      Dec 17 '18 at 8:42






    • 2




      $begingroup$
      @DavidG I appreciate that I have sparked you in favor of Ramanujan's Master Theorem. To be honest it is my favourite Theorem right now due the capability of it $($seefor example [here](math.stackexchange.com/q/3012122), [here](math.stackexchange.com/q/2937122) and [here](math.stackexchange.com/questions/2945796/….
      $endgroup$
      – mrtaurho
      Dec 17 '18 at 20:05












    • $begingroup$
      In light of spending the last week absorbed with tricky series and integrals, Ramanujan's Master Theorem seems almost too good to be true. We are certainly very lucky to have his contributions.
      $endgroup$
      – DavidG
      Dec 19 '18 at 4:22










    • $begingroup$
      @DavidG But it is true! ^^ One of the most asthonishing proofs of Ramanujan's Master Theorem for me was the one proposed by the user Count Iblis here using a technique known as "Umbral calculus". Moreover he also gave another possible proof which does not invoke the Mellin Inversion Theorem aswell.
      $endgroup$
      – mrtaurho
      Dec 19 '18 at 12:15








    4




    4




    $begingroup$
    This is my first dance with Ramanujan's Master Theorem. Only last week I was blown away learning 'Glassers Master Theorem'! now this! two mathgasms in two weeks! ..... Thanks for the post!
    $endgroup$
    – DavidG
    Dec 17 '18 at 8:42




    $begingroup$
    This is my first dance with Ramanujan's Master Theorem. Only last week I was blown away learning 'Glassers Master Theorem'! now this! two mathgasms in two weeks! ..... Thanks for the post!
    $endgroup$
    – DavidG
    Dec 17 '18 at 8:42




    2




    2




    $begingroup$
    @DavidG I appreciate that I have sparked you in favor of Ramanujan's Master Theorem. To be honest it is my favourite Theorem right now due the capability of it $($seefor example [here](math.stackexchange.com/q/3012122), [here](math.stackexchange.com/q/2937122) and [here](math.stackexchange.com/questions/2945796/….
    $endgroup$
    – mrtaurho
    Dec 17 '18 at 20:05






    $begingroup$
    @DavidG I appreciate that I have sparked you in favor of Ramanujan's Master Theorem. To be honest it is my favourite Theorem right now due the capability of it $($seefor example [here](math.stackexchange.com/q/3012122), [here](math.stackexchange.com/q/2937122) and [here](math.stackexchange.com/questions/2945796/….
    $endgroup$
    – mrtaurho
    Dec 17 '18 at 20:05














    $begingroup$
    In light of spending the last week absorbed with tricky series and integrals, Ramanujan's Master Theorem seems almost too good to be true. We are certainly very lucky to have his contributions.
    $endgroup$
    – DavidG
    Dec 19 '18 at 4:22




    $begingroup$
    In light of spending the last week absorbed with tricky series and integrals, Ramanujan's Master Theorem seems almost too good to be true. We are certainly very lucky to have his contributions.
    $endgroup$
    – DavidG
    Dec 19 '18 at 4:22












    $begingroup$
    @DavidG But it is true! ^^ One of the most asthonishing proofs of Ramanujan's Master Theorem for me was the one proposed by the user Count Iblis here using a technique known as "Umbral calculus". Moreover he also gave another possible proof which does not invoke the Mellin Inversion Theorem aswell.
    $endgroup$
    – mrtaurho
    Dec 19 '18 at 12:15




    $begingroup$
    @DavidG But it is true! ^^ One of the most asthonishing proofs of Ramanujan's Master Theorem for me was the one proposed by the user Count Iblis here using a technique known as "Umbral calculus". Moreover he also gave another possible proof which does not invoke the Mellin Inversion Theorem aswell.
    $endgroup$
    – mrtaurho
    Dec 19 '18 at 12:15











    3












    $begingroup$

    Once again I will offer up a method that first converts the integral to a double integral.



    For $r > 0$, we begin by enforcing a substitution of $x mapsto x^{1/r}$. Doing so yields
    $$I = frac{1}{r} int_0^infty frac{x^{1/r - 1}}{1 + x} , dx.$$



    Now noting that
    $$frac{1}{1 + x} = int_0^infty e^{-u(1 + x)} , du,$$
    our integral can be rewritten as
    $$I = frac{1}{r} int_0^infty x^{1/r - 1} int_0^infty e^{-u (1 + x)} , du , dx,$$
    or
    $$I = frac{1}{r} int_0^infty e^{-u} int_0^infty x^{1/r - 1} e^{-ux} , dx , du,$$
    after changing the order of integration.



    Next we enforce a substitution of $x mapsto x/u$. This gives
    begin{align}
    I &= frac{1}{r} int_0^infty u^{- 1/r} e^{-u} , du int_0^infty x^{1/r - 1} e^{-x} , dx\
    &= frac{1}{r} Gamma left (1 - frac{1}{r} right ) Gamma left (frac{1}{r} right )\
    &= frac{pi}{r sin left (frac{pi}{r} right )},
    end{align}

    where in the last line we have made use of Euler's reflexion formula for the gamma function.






    share|cite|improve this answer









    $endgroup$









    • 2




      $begingroup$
      You and your double integrals!!!!!!!! no, love it mate, thanks heaps. Great to see it used in many ways.
      $endgroup$
      – DavidG
      Dec 17 '18 at 9:26
















    3












    $begingroup$

    Once again I will offer up a method that first converts the integral to a double integral.



    For $r > 0$, we begin by enforcing a substitution of $x mapsto x^{1/r}$. Doing so yields
    $$I = frac{1}{r} int_0^infty frac{x^{1/r - 1}}{1 + x} , dx.$$



    Now noting that
    $$frac{1}{1 + x} = int_0^infty e^{-u(1 + x)} , du,$$
    our integral can be rewritten as
    $$I = frac{1}{r} int_0^infty x^{1/r - 1} int_0^infty e^{-u (1 + x)} , du , dx,$$
    or
    $$I = frac{1}{r} int_0^infty e^{-u} int_0^infty x^{1/r - 1} e^{-ux} , dx , du,$$
    after changing the order of integration.



    Next we enforce a substitution of $x mapsto x/u$. This gives
    begin{align}
    I &= frac{1}{r} int_0^infty u^{- 1/r} e^{-u} , du int_0^infty x^{1/r - 1} e^{-x} , dx\
    &= frac{1}{r} Gamma left (1 - frac{1}{r} right ) Gamma left (frac{1}{r} right )\
    &= frac{pi}{r sin left (frac{pi}{r} right )},
    end{align}

    where in the last line we have made use of Euler's reflexion formula for the gamma function.






    share|cite|improve this answer









    $endgroup$









    • 2




      $begingroup$
      You and your double integrals!!!!!!!! no, love it mate, thanks heaps. Great to see it used in many ways.
      $endgroup$
      – DavidG
      Dec 17 '18 at 9:26














    3












    3








    3





    $begingroup$

    Once again I will offer up a method that first converts the integral to a double integral.



    For $r > 0$, we begin by enforcing a substitution of $x mapsto x^{1/r}$. Doing so yields
    $$I = frac{1}{r} int_0^infty frac{x^{1/r - 1}}{1 + x} , dx.$$



    Now noting that
    $$frac{1}{1 + x} = int_0^infty e^{-u(1 + x)} , du,$$
    our integral can be rewritten as
    $$I = frac{1}{r} int_0^infty x^{1/r - 1} int_0^infty e^{-u (1 + x)} , du , dx,$$
    or
    $$I = frac{1}{r} int_0^infty e^{-u} int_0^infty x^{1/r - 1} e^{-ux} , dx , du,$$
    after changing the order of integration.



    Next we enforce a substitution of $x mapsto x/u$. This gives
    begin{align}
    I &= frac{1}{r} int_0^infty u^{- 1/r} e^{-u} , du int_0^infty x^{1/r - 1} e^{-x} , dx\
    &= frac{1}{r} Gamma left (1 - frac{1}{r} right ) Gamma left (frac{1}{r} right )\
    &= frac{pi}{r sin left (frac{pi}{r} right )},
    end{align}

    where in the last line we have made use of Euler's reflexion formula for the gamma function.






    share|cite|improve this answer









    $endgroup$



    Once again I will offer up a method that first converts the integral to a double integral.



    For $r > 0$, we begin by enforcing a substitution of $x mapsto x^{1/r}$. Doing so yields
    $$I = frac{1}{r} int_0^infty frac{x^{1/r - 1}}{1 + x} , dx.$$



    Now noting that
    $$frac{1}{1 + x} = int_0^infty e^{-u(1 + x)} , du,$$
    our integral can be rewritten as
    $$I = frac{1}{r} int_0^infty x^{1/r - 1} int_0^infty e^{-u (1 + x)} , du , dx,$$
    or
    $$I = frac{1}{r} int_0^infty e^{-u} int_0^infty x^{1/r - 1} e^{-ux} , dx , du,$$
    after changing the order of integration.



    Next we enforce a substitution of $x mapsto x/u$. This gives
    begin{align}
    I &= frac{1}{r} int_0^infty u^{- 1/r} e^{-u} , du int_0^infty x^{1/r - 1} e^{-x} , dx\
    &= frac{1}{r} Gamma left (1 - frac{1}{r} right ) Gamma left (frac{1}{r} right )\
    &= frac{pi}{r sin left (frac{pi}{r} right )},
    end{align}

    where in the last line we have made use of Euler's reflexion formula for the gamma function.







    share|cite|improve this answer












    share|cite|improve this answer



    share|cite|improve this answer










    answered Dec 17 '18 at 9:25









    omegadotomegadot

    6,3972829




    6,3972829








    • 2




      $begingroup$
      You and your double integrals!!!!!!!! no, love it mate, thanks heaps. Great to see it used in many ways.
      $endgroup$
      – DavidG
      Dec 17 '18 at 9:26














    • 2




      $begingroup$
      You and your double integrals!!!!!!!! no, love it mate, thanks heaps. Great to see it used in many ways.
      $endgroup$
      – DavidG
      Dec 17 '18 at 9:26








    2




    2




    $begingroup$
    You and your double integrals!!!!!!!! no, love it mate, thanks heaps. Great to see it used in many ways.
    $endgroup$
    – DavidG
    Dec 17 '18 at 9:26




    $begingroup$
    You and your double integrals!!!!!!!! no, love it mate, thanks heaps. Great to see it used in many ways.
    $endgroup$
    – DavidG
    Dec 17 '18 at 9:26











    3












    $begingroup$

    NOT A FULL SOLUTION:



    I've been working with special cases of the integral.



    Here we will consider $r = 2m$ where $m in mathbb{N}$. In doing so, we observe that the roots of the denominator are $m$ pairs of complex roots $(z, c(z))$ where $c(z)$ is the conjugate of $z$. To verify this:



    begin{align}
    x^{2m} + 1 = 0 rightarrow x^{2m} = e^{pi i}
    end{align}



    By De Moivre's formula, we observe that:



    begin{align}
    x = expleft({frac{pi + 2pi j}{2m} i} right) mbox{ for } j = 0dots 2m - 1
    end{align}



    Which we can express as the set



    begin{align}
    S &= Bigg{ expleft({frac{pi + 2pi cdot 0}{2m} i} right) , :expleft({frac{pi + 2pi cdot 1}{2m} i} right),dots,:expleft({frac{pi + 2pi cdot (2m - 2)}{2m} i} right)\
    &qquad:expleft({frac{pi + 2pi cdot (2m - 1)}{2m} i} right)Bigg} \
    end{align}



    Which can be expressed as the set of 2-tuples



    begin{align}
    S &= left{ left( expleft({frac{pi + 2pi j}{2m} i} right) , :expleft({frac{pi + 2pi(2m - 1 - j )}{2m} i} right)right): bigg|: j = 0 dots m - 1right}\
    & = left{ (z_j, cleft(z_jright):|: j = 0 dots m - 1 right}
    end{align}



    From here, we can factor $x^{2m} + 1$ into the form



    begin{align}
    x^{2m} + 1 &= prod_{r in S} left(x + r_jright)left(x + c(r_j)right) \
    &= prod_{r in S} left(x^2 + left(r_j + c(r_j)right)x + r_j c(r_j)right) \
    &= prod_{r in S} left(x^2 + 2Releft(r_jright)x + left|r_j right|^2right)
    end{align}



    For our case here $left|r_j right|^2 = 1$ and $Releft(r_jright) = cosleft({frac{pi + 2pi j}{2m} } right)$. Hence,



    begin{align}
    frac{1}{x^{2m} + 1} = prod_{j = 0}^{m - 1}frac{1}{ x^2 + 2cosleft({frac{pi + 2pi j}{2m} } right)x + 1}
    end{align}



    From here, to evaluate the integral we must employ Partial Fraction Decomposition:



    begin{align}
    frac{1}{x^{2m} + 1} = prod_{j = 0}^{m - 1}frac{1}{ x^2 + 2cosleft({frac{pi + 2pi j}{2m} } right)x + 1} = sum_{j = 0}^{m - 1}frac{alpha_j + beta_jx}{ x^2 + 2cosleft({frac{pi + 2pi j}{2m} } right)x + 1}
    end{align}



    And solve for $alpha_j$ and $beta_j$. Putting the coefficents to the side we can find general expressions for the integral:



    begin{align}
    frac{1}{x^{2m} + 1} = prod_{j = 0}^{m - 1}frac{1}{ x^2 + 2cosleft({frac{pi + 2pi j}{2m} } right)x + 1}
    end{align}



    From here, to evaluate the integral we must employ Partial Fraction Decomposition:



    begin{align}
    int_{0}^{infty}frac{1}{x^{2m} + 1}:dx &= int_{0}^{infty}sum_{j = 0}^{m - 1}frac{alpha_j + beta_jx}{ x^2 + 2cosleft({frac{pi + 2pi j}{2m} } right)x + 1}:dx \
    &= sum_{j = 0}^{m - 1}left[ int_{0}^{infty}frac{alpha_j}{ x^2 + 2cosleft({frac{pi + 2pi j}{2m} } right)x + 1}:dx + int_{0}^{infty}frac{beta_jx}{ x^2 + 2cosleft({frac{pi + 2pi j}{2m} } right)x + 1}:dxright] \
    &=sum_{j = 0}^{m - 1}left[ I_1 + I_2right]
    end{align}



    Evaluating each individually:
    begin{align}
    int_{0}^{infty} frac{alpha_j}{ x^2 + 2cosleft(frac{pi + 2pi j}{2m} right)x + 1}:dx &= left[ cscleft(frac{pi + 2pi j}{2m} right)arctanleft(frac{(x - 1)tanleft(frac{pi + 2pi j}{4m} right)}{x + 1} right)right]_{0}^{infty} \
    &= cscleft(frac{pi + 2pi j}{2m} right)left( frac{pi + 2pi j}{2m}right)
    end{align}



    Now if we consider the second integral on it's own we find that as a general expression the integral doesn't converge. This doesn't take away from this method, but to proceed we must solve for the unknown coefficients to proceed.






    share|cite|improve this answer











    $endgroup$


















      3












      $begingroup$

      NOT A FULL SOLUTION:



      I've been working with special cases of the integral.



      Here we will consider $r = 2m$ where $m in mathbb{N}$. In doing so, we observe that the roots of the denominator are $m$ pairs of complex roots $(z, c(z))$ where $c(z)$ is the conjugate of $z$. To verify this:



      begin{align}
      x^{2m} + 1 = 0 rightarrow x^{2m} = e^{pi i}
      end{align}



      By De Moivre's formula, we observe that:



      begin{align}
      x = expleft({frac{pi + 2pi j}{2m} i} right) mbox{ for } j = 0dots 2m - 1
      end{align}



      Which we can express as the set



      begin{align}
      S &= Bigg{ expleft({frac{pi + 2pi cdot 0}{2m} i} right) , :expleft({frac{pi + 2pi cdot 1}{2m} i} right),dots,:expleft({frac{pi + 2pi cdot (2m - 2)}{2m} i} right)\
      &qquad:expleft({frac{pi + 2pi cdot (2m - 1)}{2m} i} right)Bigg} \
      end{align}



      Which can be expressed as the set of 2-tuples



      begin{align}
      S &= left{ left( expleft({frac{pi + 2pi j}{2m} i} right) , :expleft({frac{pi + 2pi(2m - 1 - j )}{2m} i} right)right): bigg|: j = 0 dots m - 1right}\
      & = left{ (z_j, cleft(z_jright):|: j = 0 dots m - 1 right}
      end{align}



      From here, we can factor $x^{2m} + 1$ into the form



      begin{align}
      x^{2m} + 1 &= prod_{r in S} left(x + r_jright)left(x + c(r_j)right) \
      &= prod_{r in S} left(x^2 + left(r_j + c(r_j)right)x + r_j c(r_j)right) \
      &= prod_{r in S} left(x^2 + 2Releft(r_jright)x + left|r_j right|^2right)
      end{align}



      For our case here $left|r_j right|^2 = 1$ and $Releft(r_jright) = cosleft({frac{pi + 2pi j}{2m} } right)$. Hence,



      begin{align}
      frac{1}{x^{2m} + 1} = prod_{j = 0}^{m - 1}frac{1}{ x^2 + 2cosleft({frac{pi + 2pi j}{2m} } right)x + 1}
      end{align}



      From here, to evaluate the integral we must employ Partial Fraction Decomposition:



      begin{align}
      frac{1}{x^{2m} + 1} = prod_{j = 0}^{m - 1}frac{1}{ x^2 + 2cosleft({frac{pi + 2pi j}{2m} } right)x + 1} = sum_{j = 0}^{m - 1}frac{alpha_j + beta_jx}{ x^2 + 2cosleft({frac{pi + 2pi j}{2m} } right)x + 1}
      end{align}



      And solve for $alpha_j$ and $beta_j$. Putting the coefficents to the side we can find general expressions for the integral:



      begin{align}
      frac{1}{x^{2m} + 1} = prod_{j = 0}^{m - 1}frac{1}{ x^2 + 2cosleft({frac{pi + 2pi j}{2m} } right)x + 1}
      end{align}



      From here, to evaluate the integral we must employ Partial Fraction Decomposition:



      begin{align}
      int_{0}^{infty}frac{1}{x^{2m} + 1}:dx &= int_{0}^{infty}sum_{j = 0}^{m - 1}frac{alpha_j + beta_jx}{ x^2 + 2cosleft({frac{pi + 2pi j}{2m} } right)x + 1}:dx \
      &= sum_{j = 0}^{m - 1}left[ int_{0}^{infty}frac{alpha_j}{ x^2 + 2cosleft({frac{pi + 2pi j}{2m} } right)x + 1}:dx + int_{0}^{infty}frac{beta_jx}{ x^2 + 2cosleft({frac{pi + 2pi j}{2m} } right)x + 1}:dxright] \
      &=sum_{j = 0}^{m - 1}left[ I_1 + I_2right]
      end{align}



      Evaluating each individually:
      begin{align}
      int_{0}^{infty} frac{alpha_j}{ x^2 + 2cosleft(frac{pi + 2pi j}{2m} right)x + 1}:dx &= left[ cscleft(frac{pi + 2pi j}{2m} right)arctanleft(frac{(x - 1)tanleft(frac{pi + 2pi j}{4m} right)}{x + 1} right)right]_{0}^{infty} \
      &= cscleft(frac{pi + 2pi j}{2m} right)left( frac{pi + 2pi j}{2m}right)
      end{align}



      Now if we consider the second integral on it's own we find that as a general expression the integral doesn't converge. This doesn't take away from this method, but to proceed we must solve for the unknown coefficients to proceed.






      share|cite|improve this answer











      $endgroup$
















        3












        3








        3





        $begingroup$

        NOT A FULL SOLUTION:



        I've been working with special cases of the integral.



        Here we will consider $r = 2m$ where $m in mathbb{N}$. In doing so, we observe that the roots of the denominator are $m$ pairs of complex roots $(z, c(z))$ where $c(z)$ is the conjugate of $z$. To verify this:



        begin{align}
        x^{2m} + 1 = 0 rightarrow x^{2m} = e^{pi i}
        end{align}



        By De Moivre's formula, we observe that:



        begin{align}
        x = expleft({frac{pi + 2pi j}{2m} i} right) mbox{ for } j = 0dots 2m - 1
        end{align}



        Which we can express as the set



        begin{align}
        S &= Bigg{ expleft({frac{pi + 2pi cdot 0}{2m} i} right) , :expleft({frac{pi + 2pi cdot 1}{2m} i} right),dots,:expleft({frac{pi + 2pi cdot (2m - 2)}{2m} i} right)\
        &qquad:expleft({frac{pi + 2pi cdot (2m - 1)}{2m} i} right)Bigg} \
        end{align}



        Which can be expressed as the set of 2-tuples



        begin{align}
        S &= left{ left( expleft({frac{pi + 2pi j}{2m} i} right) , :expleft({frac{pi + 2pi(2m - 1 - j )}{2m} i} right)right): bigg|: j = 0 dots m - 1right}\
        & = left{ (z_j, cleft(z_jright):|: j = 0 dots m - 1 right}
        end{align}



        From here, we can factor $x^{2m} + 1$ into the form



        begin{align}
        x^{2m} + 1 &= prod_{r in S} left(x + r_jright)left(x + c(r_j)right) \
        &= prod_{r in S} left(x^2 + left(r_j + c(r_j)right)x + r_j c(r_j)right) \
        &= prod_{r in S} left(x^2 + 2Releft(r_jright)x + left|r_j right|^2right)
        end{align}



        For our case here $left|r_j right|^2 = 1$ and $Releft(r_jright) = cosleft({frac{pi + 2pi j}{2m} } right)$. Hence,



        begin{align}
        frac{1}{x^{2m} + 1} = prod_{j = 0}^{m - 1}frac{1}{ x^2 + 2cosleft({frac{pi + 2pi j}{2m} } right)x + 1}
        end{align}



        From here, to evaluate the integral we must employ Partial Fraction Decomposition:



        begin{align}
        frac{1}{x^{2m} + 1} = prod_{j = 0}^{m - 1}frac{1}{ x^2 + 2cosleft({frac{pi + 2pi j}{2m} } right)x + 1} = sum_{j = 0}^{m - 1}frac{alpha_j + beta_jx}{ x^2 + 2cosleft({frac{pi + 2pi j}{2m} } right)x + 1}
        end{align}



        And solve for $alpha_j$ and $beta_j$. Putting the coefficents to the side we can find general expressions for the integral:



        begin{align}
        frac{1}{x^{2m} + 1} = prod_{j = 0}^{m - 1}frac{1}{ x^2 + 2cosleft({frac{pi + 2pi j}{2m} } right)x + 1}
        end{align}



        From here, to evaluate the integral we must employ Partial Fraction Decomposition:



        begin{align}
        int_{0}^{infty}frac{1}{x^{2m} + 1}:dx &= int_{0}^{infty}sum_{j = 0}^{m - 1}frac{alpha_j + beta_jx}{ x^2 + 2cosleft({frac{pi + 2pi j}{2m} } right)x + 1}:dx \
        &= sum_{j = 0}^{m - 1}left[ int_{0}^{infty}frac{alpha_j}{ x^2 + 2cosleft({frac{pi + 2pi j}{2m} } right)x + 1}:dx + int_{0}^{infty}frac{beta_jx}{ x^2 + 2cosleft({frac{pi + 2pi j}{2m} } right)x + 1}:dxright] \
        &=sum_{j = 0}^{m - 1}left[ I_1 + I_2right]
        end{align}



        Evaluating each individually:
        begin{align}
        int_{0}^{infty} frac{alpha_j}{ x^2 + 2cosleft(frac{pi + 2pi j}{2m} right)x + 1}:dx &= left[ cscleft(frac{pi + 2pi j}{2m} right)arctanleft(frac{(x - 1)tanleft(frac{pi + 2pi j}{4m} right)}{x + 1} right)right]_{0}^{infty} \
        &= cscleft(frac{pi + 2pi j}{2m} right)left( frac{pi + 2pi j}{2m}right)
        end{align}



        Now if we consider the second integral on it's own we find that as a general expression the integral doesn't converge. This doesn't take away from this method, but to proceed we must solve for the unknown coefficients to proceed.






        share|cite|improve this answer











        $endgroup$



        NOT A FULL SOLUTION:



        I've been working with special cases of the integral.



        Here we will consider $r = 2m$ where $m in mathbb{N}$. In doing so, we observe that the roots of the denominator are $m$ pairs of complex roots $(z, c(z))$ where $c(z)$ is the conjugate of $z$. To verify this:



        begin{align}
        x^{2m} + 1 = 0 rightarrow x^{2m} = e^{pi i}
        end{align}



        By De Moivre's formula, we observe that:



        begin{align}
        x = expleft({frac{pi + 2pi j}{2m} i} right) mbox{ for } j = 0dots 2m - 1
        end{align}



        Which we can express as the set



        begin{align}
        S &= Bigg{ expleft({frac{pi + 2pi cdot 0}{2m} i} right) , :expleft({frac{pi + 2pi cdot 1}{2m} i} right),dots,:expleft({frac{pi + 2pi cdot (2m - 2)}{2m} i} right)\
        &qquad:expleft({frac{pi + 2pi cdot (2m - 1)}{2m} i} right)Bigg} \
        end{align}



        Which can be expressed as the set of 2-tuples



        begin{align}
        S &= left{ left( expleft({frac{pi + 2pi j}{2m} i} right) , :expleft({frac{pi + 2pi(2m - 1 - j )}{2m} i} right)right): bigg|: j = 0 dots m - 1right}\
        & = left{ (z_j, cleft(z_jright):|: j = 0 dots m - 1 right}
        end{align}



        From here, we can factor $x^{2m} + 1$ into the form



        begin{align}
        x^{2m} + 1 &= prod_{r in S} left(x + r_jright)left(x + c(r_j)right) \
        &= prod_{r in S} left(x^2 + left(r_j + c(r_j)right)x + r_j c(r_j)right) \
        &= prod_{r in S} left(x^2 + 2Releft(r_jright)x + left|r_j right|^2right)
        end{align}



        For our case here $left|r_j right|^2 = 1$ and $Releft(r_jright) = cosleft({frac{pi + 2pi j}{2m} } right)$. Hence,



        begin{align}
        frac{1}{x^{2m} + 1} = prod_{j = 0}^{m - 1}frac{1}{ x^2 + 2cosleft({frac{pi + 2pi j}{2m} } right)x + 1}
        end{align}



        From here, to evaluate the integral we must employ Partial Fraction Decomposition:



        begin{align}
        frac{1}{x^{2m} + 1} = prod_{j = 0}^{m - 1}frac{1}{ x^2 + 2cosleft({frac{pi + 2pi j}{2m} } right)x + 1} = sum_{j = 0}^{m - 1}frac{alpha_j + beta_jx}{ x^2 + 2cosleft({frac{pi + 2pi j}{2m} } right)x + 1}
        end{align}



        And solve for $alpha_j$ and $beta_j$. Putting the coefficents to the side we can find general expressions for the integral:



        begin{align}
        frac{1}{x^{2m} + 1} = prod_{j = 0}^{m - 1}frac{1}{ x^2 + 2cosleft({frac{pi + 2pi j}{2m} } right)x + 1}
        end{align}



        From here, to evaluate the integral we must employ Partial Fraction Decomposition:



        begin{align}
        int_{0}^{infty}frac{1}{x^{2m} + 1}:dx &= int_{0}^{infty}sum_{j = 0}^{m - 1}frac{alpha_j + beta_jx}{ x^2 + 2cosleft({frac{pi + 2pi j}{2m} } right)x + 1}:dx \
        &= sum_{j = 0}^{m - 1}left[ int_{0}^{infty}frac{alpha_j}{ x^2 + 2cosleft({frac{pi + 2pi j}{2m} } right)x + 1}:dx + int_{0}^{infty}frac{beta_jx}{ x^2 + 2cosleft({frac{pi + 2pi j}{2m} } right)x + 1}:dxright] \
        &=sum_{j = 0}^{m - 1}left[ I_1 + I_2right]
        end{align}



        Evaluating each individually:
        begin{align}
        int_{0}^{infty} frac{alpha_j}{ x^2 + 2cosleft(frac{pi + 2pi j}{2m} right)x + 1}:dx &= left[ cscleft(frac{pi + 2pi j}{2m} right)arctanleft(frac{(x - 1)tanleft(frac{pi + 2pi j}{4m} right)}{x + 1} right)right]_{0}^{infty} \
        &= cscleft(frac{pi + 2pi j}{2m} right)left( frac{pi + 2pi j}{2m}right)
        end{align}



        Now if we consider the second integral on it's own we find that as a general expression the integral doesn't converge. This doesn't take away from this method, but to proceed we must solve for the unknown coefficients to proceed.







        share|cite|improve this answer














        share|cite|improve this answer



        share|cite|improve this answer








        edited Dec 24 '18 at 23:19

























        answered Dec 20 '18 at 1:39









        DavidGDavidG

        2,5461726




        2,5461726















            Popular posts from this blog

            Plaza Victoria

            In PowerPoint, is there a keyboard shortcut for bulleted / numbered list?

            How to put 3 figures in Latex with 2 figures side by side and 1 below these side by side images but in...